What is the Matrix of Reflection in Euclidean Space?

Click For Summary
The discussion focuses on calculating the Matrix of Reflection in a three-dimensional Euclidean space using an orthonormal basis. The challenge lies in determining the matrix entries, with suggestions to either use the given basis vectors or create an orthonormal basis for the subspace spanned by specific vectors. A proposed method involves treating the basis vectors as standard unit vectors and applying reflection principles, where the orthogonal vector is negated. After establishing the matrix in the standard basis, a change of basis to the original orthonormal vectors can be applied if necessary. This approach aims to simplify the calculation process while ensuring accuracy in the reflection matrix.
gop
Messages
55
Reaction score
0

Homework Statement



V is a three-dimensional euclidean space and v1,v2,v3 is a orthonormal base of that space.
Calculate the Matrix of the reflection over the subspace spanned by v1+v2 and v1+2*v2+3*v3 .


Homework Equations





The Attempt at a Solution



To determine the matrix I have first to select a base I could try to use v1,v2,v3 but I can't see how to determine the entries of the matrix then.
I could use v1+v2 and v1+2*v2+3*v3 (the base of the subspace) and try to extend to a base of R^3; however I can't see how to do that with the general case without knowing what v1,v2,v3 actually is.
 
Physics news on Phys.org
Why not just write the matrix in the v1,v2,v3 basis? I.e. just treat them as though they were i,j,k. Create an orthonormal basis for the subspace. The basis vectors for it are fixed by the reflection and the orthogonal vector is multiplied by (-1). Once you have it in that basis, then if you really have to, apply the basis change from the standard basis to v1,v2,v3.
 
Question: A clock's minute hand has length 4 and its hour hand has length 3. What is the distance between the tips at the moment when it is increasing most rapidly?(Putnam Exam Question) Answer: Making assumption that both the hands moves at constant angular velocities, the answer is ## \sqrt{7} .## But don't you think this assumption is somewhat doubtful and wrong?

Similar threads

Replies
6
Views
1K
Replies
4
Views
2K
  • · Replies 1 ·
Replies
1
Views
1K
Replies
15
Views
3K
  • · Replies 8 ·
Replies
8
Views
2K
  • · Replies 7 ·
Replies
7
Views
2K
  • · Replies 1 ·
Replies
1
Views
2K
  • · Replies 1 ·
Replies
1
Views
2K
  • · Replies 5 ·
Replies
5
Views
14K
  • · Replies 6 ·
Replies
6
Views
2K